www.vorkurse.de
Ein Projekt von vorhilfe.de
Die Online-Kurse der Vorhilfe

E-Learning leicht gemacht.
Hallo Gast!einloggen | registrieren ]
Startseite · Mitglieder · Teams · Forum · Wissen · Kurse · Impressum
Forenbaum
^ Forenbaum
Status Mathe-Vorkurse
  Status Organisatorisches
  Status Schule
    Status Wiederholung Algebra
    Status Einführung Analysis
    Status Einführung Analytisc
    Status VK 21: Mathematik 6.
    Status VK 37: Kurvendiskussionen
    Status VK Abivorbereitungen
  Status Universität
    Status Lerngruppe LinAlg
    Status VK 13 Analysis I FH
    Status Algebra 2006
    Status VK 22: Algebra 2007
    Status GruMiHH 06
    Status VK 58: Algebra 1
    Status VK 59: Lineare Algebra
    Status VK 60: Analysis
    Status Wahrscheinlichkeitst

Gezeigt werden alle Foren bis zur Tiefe 2

Navigation
 Startseite...
 Neuerdings beta neu
 Forum...
 vorwissen...
 vorkurse...
 Werkzeuge...
 Nachhilfevermittlung beta...
 Online-Spiele beta
 Suchen
 Verein...
 Impressum
Das Projekt
Server und Internetanbindung werden durch Spenden finanziert.
Organisiert wird das Projekt von unserem Koordinatorenteam.
Hunderte Mitglieder helfen ehrenamtlich in unseren moderierten Foren.
Anbieter der Seite ist der gemeinnützige Verein "Vorhilfe.de e.V.".
Partnerseiten
Weitere Fächer:

Open Source FunktionenplotterFunkyPlot: Kostenloser und quelloffener Funktionenplotter für Linux und andere Betriebssysteme
Forum "Uni-Analysis-Komplexe Zahlen" - Folge komplexer Zahlen
Folge komplexer Zahlen < Komplexe Zahlen < Analysis < Hochschule < Mathe < Vorhilfe
Ansicht: [ geschachtelt ] | ^ Forum "Uni-Analysis-Komplexe Zahlen"  | ^^ Alle Foren  | ^ Forenbaum  | Materialien

Folge komplexer Zahlen: Tipp/Korrektur
Status: (Frage) beantwortet Status 
Datum: 20:02 Di 20.12.2011
Autor: Lustique

Aufgabe
Untersuchen Sie die angegebenen Folgen auf Konvergenz:

[...]

[mm]c_n=2^{-n}\left(1-\zeta_k^n\right)^n[/mm] für festes [mm]k\in\mathbb{N}_0[/mm], wobei [mm]\zeta_k=e^{i*\frac{2\pi}{k}}[/mm]




Hallo mal wieder,

ich habe ein paar Probleme mit dieser Aufgabe, und es wäre schön, wenn ihr mir da behilflich sein könntet.

1. Es müsste doch [mm]k\in\mathbb{N}[/mm] gelten, oder nicht? Ich darf doch nicht durch 0 teilen, oder?

2. Ich habe mir gedacht, ich mache da mal ein paar Fallunterscheidungen. Ich habe mir das ganze jetzt mal für [mm]k=1,2,3,4,5[/mm] angeguckt.

Für [mm]k=1[/mm] ist es ja klar, da [mm]\zeta_1=e^{i 2\pi}=1[/mm] gilt, somit also [mm]c_n=0[/mm], oder?

Für [mm]k=2[/mm] gilt ja nun [mm]\zeta_2=e^{i \frac{2\pi}{2}}=-1[/mm]. Ich habe mir dann die Teilfolgen [mm]c_{2n}[/mm] und [mm]c_{2n+1}[/mm] von [mm]c_n[/mm] angeguckt, die ja beide verschiedene Grenzwerte habe, woraus folgt, dass [mm]c_n[/mm] nicht konvergiert.

Für [mm]k=3[/mm] gab es dann aber das erste Problem, da ich nicht weiß, ob ich folgendes machen darf:
[mm]c_n=2^{-n}\left(1-\left(e^{i \frac{2\pi}{3}}\right)^n\right)^n=2^{-n}\left(1-\left(e^{i 2\pi}\right)^{\frac{n}{3}}\right)^n=2^{-n}\left(1-1^{\frac{n}{3}\right)^n=0[/mm]
WolframAlpha meint nämlich, dass [mm]e^{i \frac{2\pi}{3}}\right)=\left(e^{i 2\pi}\right)^\frac{1}{3}[/mm] falsch wäre. Außerdem kam auch in der Vorlesung, dass es für [mm]n\in\mathbb{N}[/mm] und [mm]w\in\mathbb{C}\setminus\{0\}[/mm] genau [mm]n[/mm] Lösungen der Gleichung [mm]z^n=w[/mm] gibt. Wurzeln sind also schon mal nicht eindeutig. Aber wie soll man das sonst angehen?

Darf ich so umformen? Und wie mache ich jetzt am besten genau weiter? Sehe ich das richtig, wenn ich annehme, dass [mm]\left(e^{i*\frac{2\pi}{k}}\right)^n=\left\{-1,1,i,-i\right\}[/mm] gilt? Habe ich da also eine Periodizität und reicht es, das Ganze für [mm]k=1,2,3,4[/mm] zu untersuchen?


Ist das alles kompletter Käse, außer für [mm]k=1[/mm] und [mm]k=2[/mm]?


        
Bezug
Folge komplexer Zahlen: Antwort
Status: (Antwort) fertig Status 
Datum: 23:15 Di 20.12.2011
Autor: Helbig


> Untersuchen Sie die angegebenen Folgen auf Konvergenz:
>  
> [...]
>  
> [mm]c_n=2^{-n}\left(1-\zeta_k^n\right)^n[/mm] für festes
> [mm]k\in\mathbb{N}_0[/mm], wobei [mm]\zeta_k=e^{i*\frac{2\pi}{k}}[/mm]
>  
>
>
> 1. Es müsste doch [mm]k\in\mathbb{N}[/mm] gelten, oder nicht? Ich
> darf doch nicht durch 0 teilen, oder?

Richtig!

>  
> 2. Ich habe mir gedacht, ich mache da mal ein paar
> Fallunterscheidungen. Ich habe mir das ganze jetzt mal für
> [mm]k=1,2,3,4,5[/mm] angeguckt.
>
> Für [mm]k=1[/mm] ist es ja klar, da [mm]\zeta_1=e^{i 2\pi}=1[/mm] gilt,
> somit also [mm]c_n=0[/mm], oder?

Richtig!

> Für [mm]k=2[/mm] gilt ja nun [mm]\zeta_2=e^{i \frac{2\pi}{2}}=-1[/mm]. Ich
> habe mir dann die Teilfolgen [mm]c_{2n}[/mm] und [mm]c_{2n+1}[/mm] von [mm]c_n[/mm]
> angeguckt, die ja beide verschiedene Grenzwerte habe,
> woraus folgt, dass [mm]c_n[/mm] nicht konvergiert.

Auch richtig!

> Für [mm]k=3[/mm] gab es dann aber das erste Problem, da ich nicht
> weiß, ob ich folgendes machen darf:
> [mm]c_n=2^{-n}\left(1-\left(e^{i \frac{2\pi}{3}}\right)^n\right)^n=2^{-n}\left(1-\left(e^{i 2\pi}\right)^{\frac{n}{3}}\right)^n=2^{-n}\left(1-1^{\frac{n}{3}\right)^n=0[/mm]

Dies ist falsch!  Tatsächlich ist der Realteil von
[mm] $e^{\bruch {2\pi} 3*i} [/mm] = [mm] \cos \bruch [/mm] { [mm] 2\pi} [/mm] 3 = - [mm] \bruch {\sqrt 3} [/mm] 2$, aber von [mm] $\left(e^{2 \pi i} \right)^ [/mm] {1/3} = [mm] 1^{1/3}=1$ [/mm] ist er eins.

>  
> WolframAlpha meint nämlich, dass [mm]e^{i \frac{2\pi}{3}}\right)=\left(e^{i 2\pi}\right)^\frac{1}{3}[/mm]
> falsch wäre.

Und wie recht WolframAlpha hat!

> Außerdem kam auch in der Vorlesung, dass es
> für [mm]n\in\mathbb{N}[/mm] und [mm]w\in\mathbb{C}\setminus\{0\}[/mm] genau
> [mm]n[/mm] Lösungen der Gleichung [mm]z^n=w[/mm]Eingabefehler: "\left" und "\right" müssen immer paarweise auftreten, es wurde aber ein Teil ohne Entsprechung gefunden (siehe rote Markierung)

gibt. Wurzeln sind also

> schon mal nicht eindeutig. Aber wie soll man das sonst
> angehen?

Zeige $\left|1-\zeta_k^n\right|\le 2$. Wegen der Periodizität von $e^{i*\alpha}$ nimmt $\zeta_k^n$ nur endlich viele Werte an, und nur für gerade $k$ ist auch $-1$ darunter, so daß für ungerade $k$ $\max \left\{\left|1-\zeta_k^n\right|\colon n\in\IN\}<2$ ist. Hieraus folgere, daß $(c_n)$ genau dann gegen $0$ konvergiert, wenn $k$ ungerade ist.

OK?

Gruß,
Wolfgang


Bezug
                
Bezug
Folge komplexer Zahlen: Frage (beantwortet)
Status: (Frage) beantwortet Status 
Datum: 00:48 Mi 21.12.2011
Autor: Lustique


> Richtig!

> Richtig!

> Auch richtig!

[happy]

> Und wie recht WolframAlpha hat!

Hätte ich mir eigentlich denken können, WolframAlpha hat mir bis jetzt immer gute Dienste geleistet. :-)

> > Außerdem kam auch in der Vorlesung, dass es
> > für [mm]n\in\mathbb{N}[/mm] und [mm]w\in\mathbb{C}\setminus\{0\}[/mm] genau
> > [mm]n[/mm] Lösungen der Gleichung [mm]z^n=w[/mm] gibt. Wurzeln sind also
> > schon mal nicht eindeutig. Aber wie soll man das sonst
> > angehen?
>  
> Zeige [mm]\left|1-\zeta_k^n\right|\le 2[/mm]. Wegen der
> Periodizität von [mm]e^{i*\alpha}[/mm] nimmt [mm]\zeta_k^n[/mm] nur endlich
> viele Werte an, und nur für gerade [mm]k[/mm] ist auch [mm]-1[/mm] darunter,
> so daß für ungerade [mm]k[/mm] [mm]\max \left\{\left|1-\zeta_k^n\right|\colon n\in\IN\}<2[/mm]
> ist. Hieraus folgere, daß [mm](c_n)[/mm] genau dann gegen [mm]0[/mm]
> konvergiert, wenn [mm]k[/mm] ungerade ist.
>  
> OK?
>  
> Gruß,
>  Wolfgang

Also, es gilt ja schon mal [mm] $\left\lvert e^{ix}\right\rvert [/mm] = 1$ für [mm] $x\in\matbb{R}$ [/mm] (habe ich ehrlich gesagt gar nicht dran gedacht), und in der Vorlesung hatten wir [mm] $e^{i\frac{\pi}{2}}=i$ [/mm] gezeigt und von da aus auch noch [mm] $e^{i\pi}=-1$, $e^{-i\frac{\pi}{2}}=-i$, $e^{i\frac{3}{2}\pi}=i$ [/mm] und [mm] $e^{2\pi i}=1$. [/mm]

Aber wie kann ich denn jetzt daraus schließen, dass [mm] $\left\lvert 1-\zeta_k^n\right\rvert\leqslant [/mm] 2$? Ich betrachte ja zum Einen die n-te Potenz, und zum anderen keinen Betrag.

Ich weiß zwar auch noch, dass [mm] $\exp\colon \mathbb{C}\to\mathbb{C}$ [/mm] stetig ist und das [mm] $\exp\left(z+2\pi i\right)=\exp\left(z\right)$ [/mm] für alle [mm] $z\in\mathbb{C}$ [/mm] erfüllt ist, aber was ich genau für diese Abschätzung machen muss, leuchtet mir gerade nicht so ganz ein, ehrlich gesagt.

Weiß ich dadurch schon, dass [mm] $\exp\left(x\right)$ [/mm] für [mm] $x\in\mathbb{R}$ [/mm] immer auf dieser Kreisscheibe mit Radius 1 um den Ursprung in der komplexen Ebene liegt (falls das überhaupt der Fall ist)? Aber woher weiß ich dann, dass [mm] $\left\lvert 1-\zeta_k^n\right\rvert$ [/mm] maximal wird, wenn [mm] $\zeta_k=-1$ [/mm] (dann wäre ja [mm] $\left\lvert 1-\zeta_k^n\right\rvert= [/mm] 2$)? Ich glaube ich habe mit komplexen Zahlen noch ein paar Probleme, vor allen Dingen, weil für die Dinger ja keine Ordnungsrelationen existieren, zumindest wenn man sich nicht den Betrag anguckt...

Aber auf jeden Fall schon mal danke dafür, dass du mir schon mal soweit geholfen hast. :-)




Bezug
                        
Bezug
Folge komplexer Zahlen: Antwort
Status: (Antwort) fertig Status 
Datum: 01:42 Mi 21.12.2011
Autor: Helbig


> Aber wie kann ich denn jetzt daraus schließen, dass
> [mm]\left\lvert 1-\zeta_k^n\right\rvert\leqslant 2[/mm]? Ich
> betrachte ja zum Einen die n-te Potenz, und zum anderen
> keinen Betrag.

[mm] $|1-\zeta^n_k|\le|1|+|\zeta^n_k|=1+1$ [/mm]


>
> Weiß ich dadurch schon, dass [mm]\exp\left(x\right)[/mm] für
> [mm]x\in\mathbb{R}[/mm] immer auf dieser Kreisscheibe mit Radius 1
> um den Ursprung in der komplexen Ebene liegt (falls das
> überhaupt der Fall ist)?

Nein! Sondern [mm] $e^{ix}$ [/mm] für [mm] $x\in\IR$ [/mm] liegt auf dem Einheitskreis. Und [mm] $\zeta^n_k$ [/mm] haben diese Form, liegen also auf dem Einheitskreis und haben den Betrag 1.


Aber woher weiß ich dann, dass

> [mm]\left\lvert 1-\zeta_k^n\right\rvert[/mm] maximal wird, wenn
> [mm]\zeta_k=-1[/mm] (dann wäre ja [mm]\left\lvert 1-\zeta_k^n\right\rvert= 2[/mm])?

Genau. Allgemein ist der Betrag [mm] $\le [/mm] 2$ und wird maximal, wenn er $=2$ ist.

Gruß,
Wolfgang

Bezug
                                
Bezug
Folge komplexer Zahlen: Mitteilung
Status: (Mitteilung) Reaktion unnötig Status 
Datum: 13:51 Mi 21.12.2011
Autor: Lustique


> [mm]|1-\zeta^n_k|\le|1|+|\zeta^n_k|=1+1[/mm]

Ach ja, danke. Da war ich mal wieder komplett betriebsblind. :D

> Nein! Sondern [mm]e^{ix}[/mm] für [mm]x\in\IR[/mm] liegt auf dem
> Einheitskreis. Und [mm]\zeta^n_k[/mm] haben diese Form, liegen also
> auf dem Einheitskreis und haben den Betrag 1.

Ja, den meinte ich eigentlich auch (ich weiß nicht mal genau was eine Kreisscheibe ist, ich muss wohl irgendwie den Begriff im Kopf gehabt haben), sonst wäre ja [mm] $|e^{ix}|\leqslant [/mm] 1$ und nicht [mm] $|e^{ix}|=1$. [/mm] :O

> Genau. Allgemein ist der Betrag [mm]\le 2[/mm] und wird maximal,
> wenn er [mm]=2[/mm] ist.

Ja, alles klar, danke! Ich hoffe mal, ich bekomme den Rest dann selbst hin.



Bezug
Ansicht: [ geschachtelt ] | ^ Forum "Uni-Analysis-Komplexe Zahlen"  | ^^ Alle Foren  | ^ Forenbaum  | Materialien


^ Seitenanfang ^
www.vorkurse.de
[ Startseite | Mitglieder | Teams | Forum | Wissen | Kurse | Impressum ]